4
$\begingroup$

Can you provide a proof for the following claim:

$$-\displaystyle\sum_{n=1}^{\infty}\frac{J_k(n)}{n} \cdot \ln\left(1-x^n\right)=\frac{x \cdot A_{k-1}(x)}{(1-x)^k} \quad \text{for} \quad |x| < 1 \quad \text{and} \quad k>1$$ where $J_k(n)$ is the Jordan's totient function and $A_k(x)$ is the kth Eulerian polynomial.

The first few Eulerian polynomials are: $A_1(x)=1 , A_2(x)=x+1 , A_3(x)=x^2+4x+1$ , etc.

The SageMath cell that demonstrates this claim can be found here.

$\endgroup$

1 Answer 1

6
$\begingroup$

The right hand side evaluates to $$\sum_{m=0}^{\infty} m^{k-1} x^m,$$ and so it remains to verify that the coefficient of $x^m$ in the l.h.s. is also $m^{k-1}$.

By differentiating the l.h.s., we have \begin{split} [x^m]\ \bigg(-\sum_{n\geq 1} \frac{J_k(n)}{n} \ln(1-x^n)\bigg) &= \frac1m [x^{m-1}]\ \sum_{n\geq 1} J_k(n)\frac{x^{n-1}}{1-x^n}\\ &=\frac1m \sum_{n|m} J_k(n) \\ &= m^{k-1} \end{split} as required.

$\endgroup$

Your Answer

By clicking “Post Your Answer”, you agree to our terms of service and acknowledge that you have read and understand our privacy policy and code of conduct.

Not the answer you're looking for? Browse other questions tagged or ask your own question.